Expected return

Hei … i am getting the wrong result can u please solve and paste what result you get tks An equally weighted portfolio containing stocks A, B and C has an expected return of 20%. Stock A and B have both an expected return of 15%. What is the expected return of stock C? (a) 15% (b) 20% © 25% (d) 30%

d by eyeballing it. can you show your calculation?

It would’ve been more interesting if they weren’t equally weighted! Dreary

definitely D

30% - Dinesh S

D .20 = .33(.15) + .33(.15) + .33(x) Solve for x and there you go. Actually it should be 33.33% but you get the point, it’s close enough.

i got the same 30% but its aint the right answer according to the teacher …but i think its right its publishing fault !! anyway tks everyone :-o))))

which answer did your teacher choose? and his explanation?

dennis2085, the answer is 30%, check your calculations. Dreary